Difference between revisions of "2021 Fall AMC 12A Problems/Problem 9"

Line 5: Line 5:
 
<math>\textbf{(A)}\ 2\sqrt{6} \qquad\textbf{(B)}\ 6\sqrt{6} \qquad\textbf{(C)}\ 24 \qquad\textbf{(D)}\ 48 \qquad\textbf{(E)}\ 576</math>
 
<math>\textbf{(A)}\ 2\sqrt{6} \qquad\textbf{(B)}\ 6\sqrt{6} \qquad\textbf{(C)}\ 24 \qquad\textbf{(D)}\ 48 \qquad\textbf{(E)}\ 576</math>
  
==Solution==
+
==Solution 1==
 
The surface area of this right rectangular prism is <math>2(\log_{2}x\log_{3}x+\log_{2}x\log_{4}x+\log_{3}x\log_{4}x).</math>
 
The surface area of this right rectangular prism is <math>2(\log_{2}x\log_{3}x+\log_{2}x\log_{4}x+\log_{3}x\log_{4}x).</math>
  
Line 20: Line 20:
 
\end{align*}</cmath>
 
\end{align*}</cmath>
 
~MRENTHUSIASM
 
~MRENTHUSIASM
 +
 +
== Solution 3 ==
 +
The surface area is
 +
<cmath>
 +
\begin{align*}
 +
2 \left( \log_2 x \cdot \log_3 x + \log_2 x \cdot \log_4 x + \log_3 x \cdot \log_4 x \right) .
 +
\end{align*}
 +
</cmath>
 +
 +
The volume is
 +
<cmath>
 +
\[
 +
\log_2 x \cdot \log_3 x \cdot \log_4 x .
 +
\]
 +
</cmath>
 +
 +
Hence,
 +
<cmath>
 +
\[
 +
\log_2 x \cdot \log_3 x \cdot \log_4 x
 +
= 2 \left( \log_2 x \cdot \log_3 x + \log_2 x \cdot \log_4 x + \log_3 x \cdot \log_4 x \right)
 +
.
 +
\]
 +
</cmath>
 +
 +
Dividing both sides by <math>\log_2 x \cdot \log_3 x \cdot \log_4 x</math>, we get
 +
<cmath>
 +
\begin{align*}
 +
1 & = 2 \left( \frac{1}{\log_4 x} + \frac{1}{\log_3 x} + \frac{1}{\log_2 x} \right) \\
 +
& = 2 \left( \frac{\log_a 4}{\log_a x} + \frac{\log_a 3}{\log_a x} + \frac{\log_a 2}{\log_a x} \right) \\
 +
& = 2 \frac{\log_a 4 + \log_a 3 + \log_a 2}{\log_a x} \\
 +
& = 2 \frac{\log_a \left( 4 \cdot 3 \cdot 2 \right)}{\log_a x} \\
 +
& = 2 \frac{\log_a 24}{\log_a x} \\
 +
& = \frac{\log_a 24^2}{\log_a x} \\
 +
& = \frac{\log_a 576}{\log_a x} .
 +
\end{align*}
 +
</cmath>
 +
 +
Therefore, the answer is <math>\boxed{\textbf{(E) }576}</math>.
 +
~Steven Chen (www.professorchenedu.com)
 +
  
 
==See Also==
 
==See Also==
 
{{AMC12 box|year=2021 Fall|ab=A|num-b=8|num-a=10}}
 
{{AMC12 box|year=2021 Fall|ab=A|num-b=8|num-a=10}}
 
{{MAA Notice}}
 
{{MAA Notice}}

Revision as of 21:40, 25 November 2021

Problem

A right rectangular prism whose surface area and volume are numerically equal has edge lengths $\log_{2}x, \log_{3}x,$ and $\log_{4}x.$ What is $x?$

$\textbf{(A)}\ 2\sqrt{6} \qquad\textbf{(B)}\ 6\sqrt{6} \qquad\textbf{(C)}\ 24 \qquad\textbf{(D)}\ 48 \qquad\textbf{(E)}\ 576$

Solution 1

The surface area of this right rectangular prism is $2(\log_{2}x\log_{3}x+\log_{2}x\log_{4}x+\log_{3}x\log_{4}x).$

The volume of this right rectangular prism is $\log_{2}x\log_{3}x\log_{4}x.$

Equating the numerical values of the surface area and the volume, we have \[2(\log_{2}x\log_{3}x+\log_{2}x\log_{4}x+\log_{3}x\log_{4}x)=\log_{2}x\log_{3}x\log_{4}x.\] Dividing both sides by $\log_{2}x\log_{3}x\log_{4}x,$ we get \[2\left(\frac{1}{\log_{4}x}+\frac{1}{\log_{3}x}+\frac{1}{\log_{2}x}\right)=1. \hspace{15mm} (\bigstar)\] Recall that $\log_{b}a=\frac{1}{\log_{a}b}$ and $\log_{b}\left(a^n\right)=n\log_{b}a,$ so we rewrite $(\bigstar)$ as \begin{align*} 2(\log_{x}4+\log_{x}3+\log_{x}2)&=1 \\ 2\log_{x}24&=1 \\ \log_{x}576&=1 \\ x&=\boxed{\textbf{(E)}\ 576}. \end{align*} ~MRENTHUSIASM

Solution 3

The surface area is \begin{align*} 2 \left( \log_2 x \cdot \log_3 x + \log_2 x \cdot \log_4 x + \log_3 x \cdot \log_4 x \right) . \end{align*}

The volume is \[ \log_2 x \cdot \log_3 x \cdot \log_4 x . \]

Hence, \[ \log_2 x \cdot \log_3 x \cdot \log_4 x = 2 \left( \log_2 x \cdot \log_3 x + \log_2 x \cdot \log_4 x + \log_3 x \cdot \log_4 x \right)  . \]

Dividing both sides by $\log_2 x \cdot \log_3 x \cdot \log_4 x$, we get \begin{align*} 1 & = 2 \left( \frac{1}{\log_4 x} + \frac{1}{\log_3 x} + \frac{1}{\log_2 x} \right) \\ & = 2 \left( \frac{\log_a 4}{\log_a x} + \frac{\log_a 3}{\log_a x} + \frac{\log_a 2}{\log_a x} \right) \\ & = 2 \frac{\log_a 4 + \log_a 3 + \log_a 2}{\log_a x} \\ & = 2 \frac{\log_a \left( 4 \cdot 3 \cdot 2 \right)}{\log_a x} \\ & = 2 \frac{\log_a 24}{\log_a x} \\ & = \frac{\log_a 24^2}{\log_a x} \\ & = \frac{\log_a 576}{\log_a x} . \end{align*}

Therefore, the answer is $\boxed{\textbf{(E) }576}$. ~Steven Chen (www.professorchenedu.com)


See Also

2021 Fall AMC 12A (ProblemsAnswer KeyResources)
Preceded by
Problem 8
Followed by
Problem 10
1 2 3 4 5 6 7 8 9 10 11 12 13 14 15 16 17 18 19 20 21 22 23 24 25
All AMC 12 Problems and Solutions

The problems on this page are copyrighted by the Mathematical Association of America's American Mathematics Competitions. AMC logo.png